Derive x/(3x-1) using first principles

  • Thread starter Thread starter Zipzap
  • Start date Start date
  • Tags Tags
    Derive
Click For Summary
The discussion revolves around deriving the function x/(3x-1) using first principles, with participants expressing confusion over obtaining the correct derivative form. One user consistently arrives at -1/(3x-1)^2 instead of the expected two separate terms. Another contributor suggests starting with the limit definition of the derivative and manipulating the expression to find a common denominator. They highlight that proper application of the chain rule can lead to the correct answer by simplifying the terms effectively. The conversation emphasizes the importance of careful algebraic manipulation in derivative calculations.
Zipzap
Messages
32
Reaction score
0
Ok, I am really stuck on this one. I tried using the first principles formula and everything, but I don't get two separate terms like I am supposed to in the actual derivative. I always end up with -1/(3x-1)^2 when I try. Can someone please help me out?
 
Physics news on Phys.org
Zipzap said:
Ok, I am really stuck on this one. I tried using the first principles formula and everything, but I don't get two separate terms like I am supposed to in the actual derivative. I always end up with -1/(3x-1)^2 when I try. Can someone please help me out?

Show us what you've done. It should start off something like this:
\lim_{h \to 0}\frac{\frac{x + h}{3(x + h) - 1} - \frac{x}{3x - 1}}{h}
 
I guess it is odd to end up with the right answer.
 
Well, Mark44, I have that in the beginning to. I then multiply denominators to get a common one, and I end up with something like:

{ [ (x+h)(3x-1) - x(3x + 3h -1) ] / [ (3x+3h-1)(3x-1) ] } / h

{ ( [3x^2 - x + 3hx - h] - [3x^2 + 3hx -x] ) / [ (3x+3h-1)(3x-1) ] } / h

My problem is that everything cancels out except for h, and that leaves me with only one term, which I know is incorrect. What am I doing wrong here?
 
If you write this as x*(3x-1)-1 and do the chain rule you get (3x-1)-1-x*(3x-1)-2*3 which if you put (3x-1)-2 in both denominators gives you (3x-1)/(3x-1)2-(3x)/(3x-1)2 and the 3x cancels out which gives what you got so you got the right answer.
 
Question: A clock's minute hand has length 4 and its hour hand has length 3. What is the distance between the tips at the moment when it is increasing most rapidly?(Putnam Exam Question) Answer: Making assumption that both the hands moves at constant angular velocities, the answer is ## \sqrt{7} .## But don't you think this assumption is somewhat doubtful and wrong?

Similar threads

  • · Replies 5 ·
Replies
5
Views
2K
  • · Replies 25 ·
Replies
25
Views
2K
Replies
3
Views
2K
  • · Replies 24 ·
Replies
24
Views
3K
  • · Replies 44 ·
2
Replies
44
Views
6K
  • · Replies 8 ·
Replies
8
Views
1K
  • · Replies 11 ·
Replies
11
Views
2K
  • · Replies 7 ·
Replies
7
Views
2K
  • · Replies 3 ·
Replies
3
Views
1K
  • · Replies 3 ·
Replies
3
Views
1K